subject
Mathematics, 13.07.2019 21:00 gautemalaismylife

Asap a box plot was made to represent the number of matches won by 14 participants in a tennis tournament. the box plot had the box shifted to the left so that the right tail was much longer than the left tail. based on the box plot, which conclusion is correct? the mean and median of matches won are equal. the mean of matches won is less than the median of matches won. most of the participants won many matches, but some participants won very few matches compared to the others. most of the participants won very few matches, but some participants won many matches compared to the others.

ansver
Answers: 1

Another question on Mathematics

question
Mathematics, 21.06.2019 17:20
Which of these equations, when solved, gives a different value of x than the other three? a9.1 = -0.2x + 10 b10 = 9.1 + 0.2x c10 – 0.2x = 9.1 d9.1 – 10 = 0.2x
Answers: 1
question
Mathematics, 21.06.2019 19:30
Weekly wages at a certain factory are normally distributed with a mean of $400 and a standard deviation of $50. find the probability that a worker selected at random makes between $450 and $500
Answers: 2
question
Mathematics, 21.06.2019 23:30
Ana earns 2.5% simple interest on $2,000 in her savings account. she does not make any deposits or withdrawals. after 3 years, how much interest will ana have earned on her savings account?
Answers: 1
question
Mathematics, 22.06.2019 04:30
Agatorade cooler has 36 cups of water. if a serving is 3/4 of a cup how many servings are in the cooler?
Answers: 1
You know the right answer?
Asap a box plot was made to represent the number of matches won by 14 participants in a tennis tou...
Questions
question
Chemistry, 28.01.2020 03:31
question
Mathematics, 28.01.2020 03:31
Questions on the website: 13722361